Graph Theory - Show that every graph with at most three cycles is planar












0












$begingroup$


While working through some problems concerning planarity, I encountered one asking me to prove that every graph with at most three cycles is planar. Being rather elementary in my knowledge of Graph Theory, I have tried approaching this through use of Kuratowski's Theorem:



A finite graph is planar if and only if it does not contain a sub-graph that is homeomorphic to $K_5$ or $K_{3,3}$.



However, from what I have observed $K_{3,3}$ has 10 cycles. In this way, recognizing that neither isolating a sub-graph nor subdividing said sub-graph can create new cycles, it would appear that a graph would need at least 10 cycles to be non-planar. I have been unable to identify any graph with fewer than ten cycles which proves non-planar. Any help as to what I am missing here would be greatly appreciated. Thanks!










share|cite|improve this question









$endgroup$








  • 1




    $begingroup$
    Without thinking too hard about the problem itself, one possibility is that the problem isn't asking you to prove an optimal statement. If it is true that any graph with fewer than 10 cycles is planar (I don't know yet if it is!), it is certainly also true that any graph with at most 3 cycles is planar.
    $endgroup$
    – Misha Lavrov
    Dec 10 '18 at 5:40










  • $begingroup$
    Are you sure $K_{3,3}$ has only $10$ cycles? I count $9$ cycles of length $4$ and a bunch ($6$?) of cycles of length $6$.
    $endgroup$
    – bof
    Dec 10 '18 at 5:41










  • $begingroup$
    I don't know what you're worried about. The problem is asking you to prove that every nonplanar graph has more than $3$ cycles, right? If you're allowed to use Kuratowski's theorem, then all you have to do is prove that $K_{3,3}$ and $K_5$ (and of course their subdivisions) have more than $3$ cycles. Looks like you've already done this for $K_{3,3}$ and $K_5$ should be just as easy if not easier.
    $endgroup$
    – bof
    Dec 10 '18 at 5:43










  • $begingroup$
    @MishaLavrov I was thinking along a similar line. However, after looking more carefully at the book I believe the an optimal solution is required
    $endgroup$
    – Katacka
    Dec 10 '18 at 6:14










  • $begingroup$
    @bof After recounting, I am fairly certain there are 15. I also found there to be 9 cycles of length 4, but only 5 cycles of length 6.
    $endgroup$
    – Katacka
    Dec 10 '18 at 6:23
















0












$begingroup$


While working through some problems concerning planarity, I encountered one asking me to prove that every graph with at most three cycles is planar. Being rather elementary in my knowledge of Graph Theory, I have tried approaching this through use of Kuratowski's Theorem:



A finite graph is planar if and only if it does not contain a sub-graph that is homeomorphic to $K_5$ or $K_{3,3}$.



However, from what I have observed $K_{3,3}$ has 10 cycles. In this way, recognizing that neither isolating a sub-graph nor subdividing said sub-graph can create new cycles, it would appear that a graph would need at least 10 cycles to be non-planar. I have been unable to identify any graph with fewer than ten cycles which proves non-planar. Any help as to what I am missing here would be greatly appreciated. Thanks!










share|cite|improve this question









$endgroup$








  • 1




    $begingroup$
    Without thinking too hard about the problem itself, one possibility is that the problem isn't asking you to prove an optimal statement. If it is true that any graph with fewer than 10 cycles is planar (I don't know yet if it is!), it is certainly also true that any graph with at most 3 cycles is planar.
    $endgroup$
    – Misha Lavrov
    Dec 10 '18 at 5:40










  • $begingroup$
    Are you sure $K_{3,3}$ has only $10$ cycles? I count $9$ cycles of length $4$ and a bunch ($6$?) of cycles of length $6$.
    $endgroup$
    – bof
    Dec 10 '18 at 5:41










  • $begingroup$
    I don't know what you're worried about. The problem is asking you to prove that every nonplanar graph has more than $3$ cycles, right? If you're allowed to use Kuratowski's theorem, then all you have to do is prove that $K_{3,3}$ and $K_5$ (and of course their subdivisions) have more than $3$ cycles. Looks like you've already done this for $K_{3,3}$ and $K_5$ should be just as easy if not easier.
    $endgroup$
    – bof
    Dec 10 '18 at 5:43










  • $begingroup$
    @MishaLavrov I was thinking along a similar line. However, after looking more carefully at the book I believe the an optimal solution is required
    $endgroup$
    – Katacka
    Dec 10 '18 at 6:14










  • $begingroup$
    @bof After recounting, I am fairly certain there are 15. I also found there to be 9 cycles of length 4, but only 5 cycles of length 6.
    $endgroup$
    – Katacka
    Dec 10 '18 at 6:23














0












0








0





$begingroup$


While working through some problems concerning planarity, I encountered one asking me to prove that every graph with at most three cycles is planar. Being rather elementary in my knowledge of Graph Theory, I have tried approaching this through use of Kuratowski's Theorem:



A finite graph is planar if and only if it does not contain a sub-graph that is homeomorphic to $K_5$ or $K_{3,3}$.



However, from what I have observed $K_{3,3}$ has 10 cycles. In this way, recognizing that neither isolating a sub-graph nor subdividing said sub-graph can create new cycles, it would appear that a graph would need at least 10 cycles to be non-planar. I have been unable to identify any graph with fewer than ten cycles which proves non-planar. Any help as to what I am missing here would be greatly appreciated. Thanks!










share|cite|improve this question









$endgroup$




While working through some problems concerning planarity, I encountered one asking me to prove that every graph with at most three cycles is planar. Being rather elementary in my knowledge of Graph Theory, I have tried approaching this through use of Kuratowski's Theorem:



A finite graph is planar if and only if it does not contain a sub-graph that is homeomorphic to $K_5$ or $K_{3,3}$.



However, from what I have observed $K_{3,3}$ has 10 cycles. In this way, recognizing that neither isolating a sub-graph nor subdividing said sub-graph can create new cycles, it would appear that a graph would need at least 10 cycles to be non-planar. I have been unable to identify any graph with fewer than ten cycles which proves non-planar. Any help as to what I am missing here would be greatly appreciated. Thanks!







graph-theory






share|cite|improve this question













share|cite|improve this question











share|cite|improve this question




share|cite|improve this question










asked Dec 10 '18 at 5:32









KatackaKatacka

1




1








  • 1




    $begingroup$
    Without thinking too hard about the problem itself, one possibility is that the problem isn't asking you to prove an optimal statement. If it is true that any graph with fewer than 10 cycles is planar (I don't know yet if it is!), it is certainly also true that any graph with at most 3 cycles is planar.
    $endgroup$
    – Misha Lavrov
    Dec 10 '18 at 5:40










  • $begingroup$
    Are you sure $K_{3,3}$ has only $10$ cycles? I count $9$ cycles of length $4$ and a bunch ($6$?) of cycles of length $6$.
    $endgroup$
    – bof
    Dec 10 '18 at 5:41










  • $begingroup$
    I don't know what you're worried about. The problem is asking you to prove that every nonplanar graph has more than $3$ cycles, right? If you're allowed to use Kuratowski's theorem, then all you have to do is prove that $K_{3,3}$ and $K_5$ (and of course their subdivisions) have more than $3$ cycles. Looks like you've already done this for $K_{3,3}$ and $K_5$ should be just as easy if not easier.
    $endgroup$
    – bof
    Dec 10 '18 at 5:43










  • $begingroup$
    @MishaLavrov I was thinking along a similar line. However, after looking more carefully at the book I believe the an optimal solution is required
    $endgroup$
    – Katacka
    Dec 10 '18 at 6:14










  • $begingroup$
    @bof After recounting, I am fairly certain there are 15. I also found there to be 9 cycles of length 4, but only 5 cycles of length 6.
    $endgroup$
    – Katacka
    Dec 10 '18 at 6:23














  • 1




    $begingroup$
    Without thinking too hard about the problem itself, one possibility is that the problem isn't asking you to prove an optimal statement. If it is true that any graph with fewer than 10 cycles is planar (I don't know yet if it is!), it is certainly also true that any graph with at most 3 cycles is planar.
    $endgroup$
    – Misha Lavrov
    Dec 10 '18 at 5:40










  • $begingroup$
    Are you sure $K_{3,3}$ has only $10$ cycles? I count $9$ cycles of length $4$ and a bunch ($6$?) of cycles of length $6$.
    $endgroup$
    – bof
    Dec 10 '18 at 5:41










  • $begingroup$
    I don't know what you're worried about. The problem is asking you to prove that every nonplanar graph has more than $3$ cycles, right? If you're allowed to use Kuratowski's theorem, then all you have to do is prove that $K_{3,3}$ and $K_5$ (and of course their subdivisions) have more than $3$ cycles. Looks like you've already done this for $K_{3,3}$ and $K_5$ should be just as easy if not easier.
    $endgroup$
    – bof
    Dec 10 '18 at 5:43










  • $begingroup$
    @MishaLavrov I was thinking along a similar line. However, after looking more carefully at the book I believe the an optimal solution is required
    $endgroup$
    – Katacka
    Dec 10 '18 at 6:14










  • $begingroup$
    @bof After recounting, I am fairly certain there are 15. I also found there to be 9 cycles of length 4, but only 5 cycles of length 6.
    $endgroup$
    – Katacka
    Dec 10 '18 at 6:23








1




1




$begingroup$
Without thinking too hard about the problem itself, one possibility is that the problem isn't asking you to prove an optimal statement. If it is true that any graph with fewer than 10 cycles is planar (I don't know yet if it is!), it is certainly also true that any graph with at most 3 cycles is planar.
$endgroup$
– Misha Lavrov
Dec 10 '18 at 5:40




$begingroup$
Without thinking too hard about the problem itself, one possibility is that the problem isn't asking you to prove an optimal statement. If it is true that any graph with fewer than 10 cycles is planar (I don't know yet if it is!), it is certainly also true that any graph with at most 3 cycles is planar.
$endgroup$
– Misha Lavrov
Dec 10 '18 at 5:40












$begingroup$
Are you sure $K_{3,3}$ has only $10$ cycles? I count $9$ cycles of length $4$ and a bunch ($6$?) of cycles of length $6$.
$endgroup$
– bof
Dec 10 '18 at 5:41




$begingroup$
Are you sure $K_{3,3}$ has only $10$ cycles? I count $9$ cycles of length $4$ and a bunch ($6$?) of cycles of length $6$.
$endgroup$
– bof
Dec 10 '18 at 5:41












$begingroup$
I don't know what you're worried about. The problem is asking you to prove that every nonplanar graph has more than $3$ cycles, right? If you're allowed to use Kuratowski's theorem, then all you have to do is prove that $K_{3,3}$ and $K_5$ (and of course their subdivisions) have more than $3$ cycles. Looks like you've already done this for $K_{3,3}$ and $K_5$ should be just as easy if not easier.
$endgroup$
– bof
Dec 10 '18 at 5:43




$begingroup$
I don't know what you're worried about. The problem is asking you to prove that every nonplanar graph has more than $3$ cycles, right? If you're allowed to use Kuratowski's theorem, then all you have to do is prove that $K_{3,3}$ and $K_5$ (and of course their subdivisions) have more than $3$ cycles. Looks like you've already done this for $K_{3,3}$ and $K_5$ should be just as easy if not easier.
$endgroup$
– bof
Dec 10 '18 at 5:43












$begingroup$
@MishaLavrov I was thinking along a similar line. However, after looking more carefully at the book I believe the an optimal solution is required
$endgroup$
– Katacka
Dec 10 '18 at 6:14




$begingroup$
@MishaLavrov I was thinking along a similar line. However, after looking more carefully at the book I believe the an optimal solution is required
$endgroup$
– Katacka
Dec 10 '18 at 6:14












$begingroup$
@bof After recounting, I am fairly certain there are 15. I also found there to be 9 cycles of length 4, but only 5 cycles of length 6.
$endgroup$
– Katacka
Dec 10 '18 at 6:23




$begingroup$
@bof After recounting, I am fairly certain there are 15. I also found there to be 9 cycles of length 4, but only 5 cycles of length 6.
$endgroup$
– Katacka
Dec 10 '18 at 6:23










0






active

oldest

votes











Your Answer





StackExchange.ifUsing("editor", function () {
return StackExchange.using("mathjaxEditing", function () {
StackExchange.MarkdownEditor.creationCallbacks.add(function (editor, postfix) {
StackExchange.mathjaxEditing.prepareWmdForMathJax(editor, postfix, [["$", "$"], ["\\(","\\)"]]);
});
});
}, "mathjax-editing");

StackExchange.ready(function() {
var channelOptions = {
tags: "".split(" "),
id: "69"
};
initTagRenderer("".split(" "), "".split(" "), channelOptions);

StackExchange.using("externalEditor", function() {
// Have to fire editor after snippets, if snippets enabled
if (StackExchange.settings.snippets.snippetsEnabled) {
StackExchange.using("snippets", function() {
createEditor();
});
}
else {
createEditor();
}
});

function createEditor() {
StackExchange.prepareEditor({
heartbeatType: 'answer',
autoActivateHeartbeat: false,
convertImagesToLinks: true,
noModals: true,
showLowRepImageUploadWarning: true,
reputationToPostImages: 10,
bindNavPrevention: true,
postfix: "",
imageUploader: {
brandingHtml: "Powered by u003ca class="icon-imgur-white" href="https://imgur.com/"u003eu003c/au003e",
contentPolicyHtml: "User contributions licensed under u003ca href="https://creativecommons.org/licenses/by-sa/3.0/"u003ecc by-sa 3.0 with attribution requiredu003c/au003e u003ca href="https://stackoverflow.com/legal/content-policy"u003e(content policy)u003c/au003e",
allowUrls: true
},
noCode: true, onDemand: true,
discardSelector: ".discard-answer"
,immediatelyShowMarkdownHelp:true
});


}
});














draft saved

draft discarded


















StackExchange.ready(
function () {
StackExchange.openid.initPostLogin('.new-post-login', 'https%3a%2f%2fmath.stackexchange.com%2fquestions%2f3033494%2fgraph-theory-show-that-every-graph-with-at-most-three-cycles-is-planar%23new-answer', 'question_page');
}
);

Post as a guest















Required, but never shown

























0






active

oldest

votes








0






active

oldest

votes









active

oldest

votes






active

oldest

votes
















draft saved

draft discarded




















































Thanks for contributing an answer to Mathematics Stack Exchange!


  • Please be sure to answer the question. Provide details and share your research!

But avoid



  • Asking for help, clarification, or responding to other answers.

  • Making statements based on opinion; back them up with references or personal experience.


Use MathJax to format equations. MathJax reference.


To learn more, see our tips on writing great answers.




draft saved


draft discarded














StackExchange.ready(
function () {
StackExchange.openid.initPostLogin('.new-post-login', 'https%3a%2f%2fmath.stackexchange.com%2fquestions%2f3033494%2fgraph-theory-show-that-every-graph-with-at-most-three-cycles-is-planar%23new-answer', 'question_page');
}
);

Post as a guest















Required, but never shown





















































Required, but never shown














Required, but never shown












Required, but never shown







Required, but never shown

































Required, but never shown














Required, but never shown












Required, but never shown







Required, but never shown







Popular posts from this blog

Plaza Victoria

In PowerPoint, is there a keyboard shortcut for bulleted / numbered list?

How to put 3 figures in Latex with 2 figures side by side and 1 below these side by side images but in...